Suppose that the speed at which cars go on the freeway is normally distributed with mean 65 mph and
standard deviation 6 miles per hour. Let X be the speed for a randomly selected car
b. If one car is randomly chosen, find the probability that it is traveling more than 63 mph.

Answers

Answer 1

Answer:

B) the probability that is traveling more than 63 mph is 0.6293 (or 62.93%)

Step-by-step explanation:

Given:

Normally DistributedMean (μ) = 65 mphStandard Deviation (σ) = 6 miles per hour

Finding the Probability:

If one car is randomly chosen, we want the probability that is traveling more than 63 mph is,

P(X > 63)

To find the value of z,

z = x - μ / σ

z is the standard scorex is the observed valueμ is the mean of the sampleσ is the standard deviation of the sample

z = 63 - 65 / 6

z = -2 / 6

z = -1 / 3 which is approximately -0.33

Using Z table (attached below):

z = -0.33

to find this on the table

on the vertical side under z go to -0.3on the horizontal next to z, go to .03

The area under the curve is 0.3707

P(z > 63) = 1 - P(z < 63)

               = 1 - 0.3707

               = 0.6293

Hence the probability that is traveling more than 63 mph is 0.6293

Learn more Probability from a similar example: https://brainly.com/question/15565069

Suppose That The Speed At Which Cars Go On The Freeway Is Normally Distributed With Mean 65 Mph Andstandard

Related Questions

!!!!!!!!!!!!!!!!!!!!!!!!!!!!!!!!!!!!!!!!!!!!!!

Answers

[tex]\quad \huge \quad \quad \boxed{ \tt \:Answer }[/tex]

[tex]\qquad \tt \rightarrow \:f(4)= -1 [/tex]

____________________________________

[tex] \large \tt Solution \: : [/tex]

If a point point (x , y) satifys a function, then it can be written as :

[tex]\qquad \tt \rightarrow \: f(x) = y[/tex]

Here, point (x , y) is replaced by (4 , -1)

[tex]\qquad \tt \rightarrow \: f(4) = - 1[/tex]

Answered by : ❝ AǫᴜᴀWɪᴢ ❞

Question 3, please help. Very urgent and due soon!

Answers

Answer:

A = 5, B = 9, C = 6

Step-by-step explanation:

let me know if you want an explanation :))

The following graph portrays the distribution of the number of spicy chicken sandwiches sold at a nearby Wendy's for the last 141 days.
The mean number of sandwiches sold per day is 91.9 and the standard deviation is 4.67. (Round your answers to 2 decimal places.)

If we use the Empirical Rule, on 68 percent of the days sales will be between

95 percent of the days sales will be between

Answers

68% of the day's sales will be between 87.23 and 96.57 and on 95% of the days, sales will be between 82.56 and 101.24.

What is the empirical rule?

If your distribution follows a normal distribution, the standard deviation and mean can tell you where the majority of the data are.

We have:

The mean number of sandwiches sold per day is 91.9 and the standard deviation is 4.67.

As we know, from the empirical rule, 68% of the data will lie in one standard deviation of the mean, i.e., in the interval with the endpoints x±s

for samples and with endpoints u±б for the population.

x = 91.9

s = 4.67

x - s = 91.9 - 4.67 = 87.23

x + s = 91.9 + 4.67 = 96.57

68% of the day's sales will be between 87.23 and 96.57

95% of the data will lie within two standard deviations of the mean, which means in the interval with the endpoints  x±2s for samples and with endpoints  u±б population.

x - 2s = 82.56

x + 2s = 101.24

On 95% of the days, sales will be between 82.56 and 101.24

Thus, 68% of the day's sales will be between 87.23 and 96.57 and on 95% of the days, sales will be between 82.56 and 101.24.

Learn more about the empirical rules here:

brainly.com/question/5440679

#SPJ1

What is the simplified form of 12x/900x²y4z6?
O 30y²|xz³|
O 30x²y2z4
O 360xy²|xz³|
O 360xy²|z³|

Answers

Answer:

[tex]360xy^2|xz^3|[/tex]

Step-by-step explanation:

Given expression:

[tex]12x \sqrt{900x^2y^4z^6}[/tex]

[tex]\textsf{Apply radical rule} \quad \sqrt{ab}=\sqrt{a}\sqrt{b}:[/tex]

[tex]\implies 12x \sqrt{900}\sqrt{x^2}\sqrt{y^4}\sqrt{z^6}[/tex]

Replace 900 with 30² :

[tex]\implies 12x \sqrt{30^2}\sqrt{x^2}\sqrt{y^4}\sqrt{z^6}[/tex]

[tex]\textsf{Apply radical rule} \quad \sqrt{a^2}=a, \quad a \geq 0:[/tex]

[tex]\implies 12x \cdot 30|x|\sqrt{y^4}\sqrt{z^6}[/tex]

[tex]\implies 360x|x|\sqrt{y^4}\sqrt{z^6}[/tex]

(We need to use the absolute value of √x² since the x term was originally to the power of 2, which means the value of x² is always positive since the exponent is even).

[tex]\textsf{Apply exponent rule} \quad \sqrt{a^m}=a^{\frac{m}{2}}:[/tex]

[tex]\implies 360x|x|\cdot y^{\frac{4}{2}}\cdot z^{\frac{6}{2}}[/tex]

Simplify:

[tex]\implies 360xy^2|xz^3|[/tex]

(We need to use the absolute value of z³ since the z term was original to the power of 6, which means the value of z⁶ is always positive since the exponent is even).

From this diagram, select the
pair of lines that must be
parallel if 24 = 23. If there
is no pair of lines, select
"none."

Answers

O is parallel to q

let's take the angle opposite to angle 4 is alpha

angle 4 = angle alpha ( vertically opposite angles)

angle alpha = angle 3 ( as angle 4 = angle 3).

hence o is parallel to q as both the angles are equal

Given the function f(x) = -5|x + 1| + 3, for what values of x is f(x) = -12? O X= -2. x = 1 O x= -2 x = 4 O x = 2. x = 4 O x = 2, x = 4

Answers

The values are x= -2 and x =4 of the given function f (x) =-5|x+1| +3

What is function?

Function is an expression, rule, or law that defines a relationship between one variable (the independent variable) and another variable (the dependent variable).

According to the question,

The given function is

[tex]f(x) = -5|x + 1| + 3[/tex]

Here f(x) = -12

Substitute f(x) and solve for x

[tex]- 12 = -5|x + 1| + 3[/tex]

By subtracting 3 on both sides

[tex]- 12 - 3 = -5|x + 1| + 3 - 3[/tex]

[tex]- 15 = -5|x + 1|[/tex]

By dividing -5 on both sides

[tex]3 = |x + 1|[/tex]

We get absolute values for the function.

[tex]x + 1 = 3 , x + 1 = -3[/tex]

By subtracting 1 on both sides

[tex]x + 1 - 1 = 3 - 1 = 2\\\\x + 1 - 1 = - 3 - 1 = -4\\\\x = 2 , x = - 4[/tex]

Therefore, the values of x are 2 and - 4 of the given function f (x) =-5|x+1| +3

Learn more about function here: https://brainly.com/question/12431044

#SPJ1

. Find the general solution to
d²y/dx²= 49y. Enter your answer as an equation y = ..

Note: d²y/dx²=k^2y

Answers

The given differential equation is linear with constant coefficients,

[tex]\dfrac{d^2y}{dx^2} - 49y = 0[/tex]

with characteristic equation

[tex]r^2 - 49 = 0[/tex]

and hence characteristic roots [tex]r=\pm7[/tex]. This means the general solution to the ODE is

[tex]y = C_1 e^{7x} + C_2 e^{-7x}[/tex]

In fact, you're given the solution already,

[tex]y = C_1 e^{kx} + C_2 e^{-kx}[/tex]

and you've determined that

[tex]\dfrac{d^2y}{dx^2} = k^2 (C_1 e^{kx} + C_2 e^{-kx}) = k^2 y[/tex]

Comparing this to the given ODE, it's obvious that [tex]k=7[/tex], so you can just replace [tex]k[/tex] with 7 in the given template solution.

Jordan wrote the expression 850p to represent the following scenario.

The cost of renting a bowling alley for a party is $850. The price per person depends on how many people attend the party. Write an expression for the price per person if p people attend.

Explain why Jordan's expression is incorrect. Provide the correct expression that represents the scenario.

Answers

Using proportions, the expression for the price per person is:

[tex]P = \frac{850}{p}[/tex]

Jordan's expression is wrong because the price per person is inverse proportional to the number of people in the party, not direct proportional.

What is a proportion?

A proportion is a fraction of a total amount, and the measures are related using a rule of three.

The cost of renting a bowling alley for a party is $850, hence the cost per person is given by:

[tex]P = \frac{850}{p}[/tex]

Jordan's expression is wrong because the price per person is inverse proportional(that is, the amount is divided by p) to the number of people in the party, not direct proportional.

More can be learned about proportions at https://brainly.com/question/24372153

#SPJ1

This is part A to one of my questions to help answer the second one, the second question is posted if anyone can help

Answers

Answer:

Step-by-step explanation:

Part A: Since 400 is being multiplied 1.06^t, for it to be equal to 399 the value 1.06^t must be less than 1. The lowest value for t given the domain restriction is 0 which would only result in 1 which will given the initial amount of 400. t would need to be less than 0 for 1.06^t to start going below the value 1 but since there is a domain restriction since t represents time, that value cannot go below 400 to reach 399.

A decision at the margin Crystal is a hard-working college senior. One Thursday, she decides to work nonstop until she has answered 100 practice problems for her math course. She starts work at 8:00 AM and uses a table to keep track of her progress throughout the day. She notices that as she gets tired, it takes her longer to solve each problem. Time Total Problems Answered 8:00 AM 0 9:00 AM 40 10:00 AM 70 11:00 AM 90 Noon 100 Use the table to answer the following questions. The marginal, or additional, gain from Crystal’s first hour of work, from 8:00 AM to 9:00 AM, is problems. The marginal gain from Crystal’s third hour of work, from 10:00 AM to 11:00 AM, is problems. Later, the teaching assistant in Crystal’s math course gives her some advice. “Based on past experience,” the teaching assistant says, “working on 35 problems raises a student’s exam score by about the same amount as reading the textbook for 1 hour.” For simplicity, assume students always cover the same number of pages during each hour they spend reading. Given this information, in order to use her 4 hours of study time to get the best exam score possible, how many hours should she have spent working on problems, and how many should she have spent reading? 0 hours working on problems, 4 hours reading 1 hour working on problems, 3 hours reading 3 hours working on problems, 1 hour reading 4 hours working on problems, 0 hours reading

Answers

She should spend 1 hour working on problems , 3 hour reading , Option B is the right answer.

What is Margin gain ?

The gain of the desired effect with respect to the previous gain is called margin gain

The marginal gain from 8:00 am to 9:00 am is

(40-0) = 40 problems

The marginal gain from 9:00 am to 10:00 am is

= 70-40 = 30 problems

The marginal gain from 10:00 am to 11:00 am is

(90-70) = 20 problems

As if she does more than one hour of problem solving her productivity decreases to 30 which is less than 1 hour of reading textbook

Therefore she should not spend more than 1 hour on problem solving.

Therefore Option B is the right answer.

1 hour working on problems , 3 hour reading

To know more about Margin gain

https://brainly.com/question/17464267

#SPJ1

Questions in the picture!

Can I have the answer to2 decimal place

__.__cm2

Answers

Answer:

16П or approximately 50.3)

Step-by-step explanation:

The area of a circle is equal to П*r²

Since the circle is inside a rectangle with side that is equal to 8, we know that d = 8

2r = d

r = 8/2

r = 4

A = П*r²

A = П*16

А = 16П ( or 16*3.141592653589793238... = 50.26548... ≈ 50.3)

Answer:

area = 50.27 [tex]cm^2[/tex]

Step-by-step explanation:

In the diagram, we see that the diameter of the circle is 8cm. Therefore the radius of the circle is 8/2 = 4cm.

[tex]area \hspace{0.15cm}of \hspace{0.15cm}circle = $\pi$r^2[/tex]

                        = π [tex](4)^2[/tex]

                        = 50.27 [tex]cm^2[/tex]

The image of a point K(1,2) after translation is K¹ (-1,2). what is the coordinate of the point R whose image is R¹ (-3,3) after undergoing the same translation. ​

Answers

Sorry big bro people do this a lot 24

find x and y please please help

Answers

L1 and L2 are parallel

so the interior alternate angles are equal

4y - 40 = 3y ( interior alternate angles)

4y - 40 = x + 15 ( vertically opposite angles).

solving the first equation, we get

4y - 3y = 40

y = 40°

putting values of y= 40° in eq. 2, we get

4y - 40= x + 15

4(40) - 40 = x + 15

160 -40 = x + 15

120 - 15 = x

105° = x

X = 105° , Y = 40°

Need the answer ASAP!!!

Answers

Answer:

  y ≈ 15272.9(1.06306^x) . . . . from regression tool

  y = 15300(1.0624^x) . . . . calculated "by hand"

Step-by-step explanation:

An exponential model is one that uses the exponential function ...

  f(x) = a·b^x

to model the data. The value of 'a' is the function value when x=0, the initial value. The value of 'b' is the growth factor, the multiplier between function values for successive values of x.

__

tool

An exponential regression tool makes short work of this. Such tools are available on some calculators, apps, and spreadsheets. All that is required is entering the data into a table and invoking the tool. The one shown in the attachment tells you the model is approximately ...

  y ≈ 15272.9(1.06306^x)

__

averaging

We can average the growth rate factors a couple of different ways. When we think of "averaging," we usually think of the arithmetic mean, the sum of values divided by their number. If we average the growth rates in this way, we get ...

  average rate of growth =

  ((161/153) +(173/161) +(184/173) +(196/184) +(207/196) +(220/207))/6 ≈ 1.0624

Using the first table value as the initial value, the model formed this way is ...

  y = 15300(1.0624^x)

__

For exponential growth it may make more sense to compute the geometric mean. This is the n-th root of the product of n numbers. In this case, it reduces to the 6-th root of the ratio of the last to the first:

  average rate of growth = (220/153)^(1/6) ≈ 1.0624

This gives the same growth factor as above, hence the same model.

The second attachment shows the application of this model to the domain used in the table. We see there are some minor differences when rounding the function value to the nearest hundred.

The differences in our two models seem to arise from rounding the data values in the table to the nearest hundred.

__

summary

The values given in the table can be used in different ways to arrive at two similar, but different exponential models of the table data:

y = 15272.9(1.06306^x)y = 15300(1.0624^x)

Interestingly, the function calculated "by hand" has a very slightly better correlation with the table data. The difference in correlation coefficients is in the 6th decimal place.

Some identical cylinders each have a radius of
5 cm and a length of 4 cm.
Rufus has enough paint to cover 2500 cm².
How many of these cylinders can Rufus paint
completely?

Answers

Answer:

  8 cylinders

Step-by-step explanation:

The surface area of a cylinder is given by the formula ...

  A = 2πr(r +l) . . . . . for radius r and length l

__

The cylinders Rufus has each have an area of ...

  A = 2π(5 cm)(5 cm +4 cm) = 90π cm² ≈ 282.74 cm²

That means 2500 cm² of paint will cover ...

  2500 cm²/(282.74 cm²/cylinder) ≈ 8.84 cylinders

Rufus has enough paint to cover 8 cylinders completely.

The graphs below have the same shape. The equation of the blue graph is F(x) = x^3. What is the equation of the red graph?

Answers

Answer:

g(x) = x³ - 2

(option B)

Step-by-step explanation:

because the shape of the graph has not changed, we know that there was no direct change to x

(this would have looked like 2x³ instead of x³)

and because the graph was not shifted along the x-axis (left or right), we know that the change did not happen inside the parenthesis

(this would have looked like (x³ - 3) instead of (x³) )

we know that the only change that has happened to this graph was a downward shift along the y-axis  which can be expressed as

g(x) = x³ - 2

The equation of the red graph will be g(x) = x³ - 2.Option B is correct.

What is the equation?

A mathematical statement consisting of an equal symbol between two algebraic expressions with the same value is known as an equation.

We know that there was no immediate change to x since the graph's form has not altered. Instead of , this would have appeared as 2x³.

Thus we can be confident that the change did not occur inside the parenthesis since the graph was not moved down the x-axis.

We may infer that this graph's sole modification was a downward shift along the y-axis, which is written as;

g(x) = x³ - 2

The graph is also attached in the attachment.

The equation of the red graph will be g(x) = x³ - 2.

Hence option B is correct.

To learn more, about equations, refer;

https://brainly.com/question/10413253

#SPJ1

Daniel plays a shooting game in a funfair. He needs 48 points to get a skateboard. Each shot that hits the target will be given 6 points while 4 points will be deducted for every failed attempt. Daniel has already failed in three attempts. how many shots that are needed to hit the target for him to win the skateboard​

Answers

Answer:

here you go

Step-by-step explanation:

no of times he failed = -3

points he losed = -3 x 4 = -12

no of times he need to hit the target to get 48 points = 6x

equation

-12 + 6x = 48

6x = 60

x = 10

no of points he  will  gain = 10 x 6 = 60

How do I solve this problem? It’s been awhile since I’ve done math

C=(2+y)h

solve for y

Answers

Answer:

y = C/h - 2

Step-by-step explanation:

Equation

C = (2 + y)*h

Solution

C = (2 + y)*h                          Divide by h

C/h = (2 + y)*h/h

C/h = 2 + y                            Subtract 2 from both sides

C/h-2 = 2-2+ y                      Combine

C/h - 2 = y

What is the equation of the line that has a slope of -4 and passes through the point (2, 3)?

Answers

If [tex]m=-4[/tex]

and a point is [tex](2,3)[/tex]

[tex]y=mx+b[/tex]

[tex]3=(-4)(2)+b[/tex] using the point and the slope

[tex]3=-8+b[/tex]

[tex]b=8+3[/tex]

[tex]b=11[/tex]

Therefore, the equation of the line will be [tex]y=-4x+11[/tex]

which number property is illustrated in the Identy ?

(1/2+1/3)+1/4 = 1/2 + (1/3+ 1/4

A) Associative
B) commutative
C) Distributive
D) Identity

Answers

The property of this equation is associative property

Houses in an real estate are all identical however a person can purchase a new house with some all or none of a set of options as indicated by the set below
how many options are there for purchasing a house in this community

{ pool , alarm , lake view , balcony }

Answers

Considering the number of subsets of the set of options, it is found that there are 16 options for purchasing a house in this community.

What is the number os subsets of a set of cardinality n?

The number of subsets is given by:

[tex]N_s = 2^n [/tex]

The set of options is given by:

O = { pool , alarm , lake view , balcony }

Which has cardinality n = 4.

The number of options for the purchased home is the number of subsets of set O, hence:

[tex]N_s = 2^4 = 16[/tex]

More can be learned about the number of subsets of a set at https://brainly.com/question/1462590

#SPJ1

what is the value of the rational expression below when is equal to 5?

Answers

Answer:

[tex]\boxed {C. -2}[/tex]

Step-by-step explanation:

[tex]\textsf {Substitute x = 5 in the expression :}[/tex]

[tex]\longrightarrow \mathsf{\frac{15 - x}{x - 10}}[/tex]

[tex]\longrightarrow \mathsf{\frac{15 - 5}{5 - 10}}[/tex]

[tex]\longrightarrow \mathsf{\frac{10}{-5}}[/tex]

[tex]\longrightarrow \mathsf{-2}[/tex]

Which ordered pair is on the graph of the equation 4x + 3y = 13?


(1, 6)


(0, –3)


(1, –3)


(4, –1)

Answers

(4,-1) is correct
There are two ways to find out answers
1) try substitute those x, y points in the equation and the number which results 13 will be the right answer.
2) u can get equation of y=13/3 - 14x/3
then u go try substitute the value of x to any number find possible solutions.

Hopes this helps.

NEED HELP ASAP WILL MARK BRAINLIEST!

Answers

Answer:

[tex]\boxed {1)log_{b}(75) = 4.317}[/tex]

[tex]\boxed {2)ln(20) = 2.9957}[/tex]

Step-by-step explanation:

[tex]\textsf {Question l :}[/tex]

[tex]\longrightarrow \mathsf {log_{b}(3) = 1.099}[/tex]

[tex]\longrightarrow \mathsf {log_{b}(5) = 1.609}[/tex]

[tex]\textsf {Identities applied :}[/tex]

[tex]\boxed {log(ab) = loga + logb}[/tex]

[tex]\boxed {log(a)^{x} = xloga}[/tex]

[tex]\textsf {We can rewrite the problem as :}[/tex]

[tex]\longrightarrow \mathsf {log_{b}(75)}[/tex]

[tex]\longrightarrow \mathsf {log_{b}(25 \times 3)}[/tex]

[tex]\longrightarrow \mathsf {log_{b}(5^{2} \times 3)}[/tex]

[tex]\longrightarrow \mathsf {log_{b}(5)^{2} + log_{b}(3)}[/tex]

[tex]\longrightarrow \mathsf {2log_{b}(5) + log_{b}(3)}[/tex]

[tex]\textsf {Now, substitute the values :}[/tex]

[tex]\longrightarrow \mathsf {2(1.609) + (1.099)}[/tex]

[tex]\longrightarrow \mathsf {3.218 + 1.099}[/tex]

[tex]\longrightarrow \mathsf {4.317}[/tex]

[tex]\boxed {log_{b}(75) = 4.317}[/tex]

[tex]\textsf {Question ll :}[/tex]

[tex]\longrightarrow \mathsf {ln(4) = 1.3863}[/tex]

[tex]\longrightarrow \mathsf {ln(5) = 1.6094}[/tex]

[tex]\textsf {Rewriting the problem :}[/tex]

[tex]\longrightarrow \mathsf {ln(20)}[/tex]

[tex]\longrightarrow \mathsf {ln(4 \times 5)}[/tex]

[tex]\longrightarrow \mathsf {ln(4) + ln(5)}[/tex]

[tex]\longrightarrow \mathsf {1.3863 + 1.6094}[/tex]

[tex]\longrightarrow \mathsf {2.9957}[/tex]

[tex]\boxed {ln(20) = 2.9957}[/tex]

Answer:

[tex]\sf \log_b(75)=4.317[/tex]

[tex]\sf \ln (20)=2.9957[/tex]

Step-by-step explanation:

Question 1

Given:

  [tex]\sf \log_b(3)=1.099[/tex]

  [tex]\sf \log_b(5)=1.609[/tex]

To evaluate [tex]\sf \log_b(75)[/tex],  replace 75 with (5 × 5 × 3):

[tex]\implies \sf \log_b(5 \cdot 5 \cdot 3)[/tex]

[tex]\textsf{Apply the Product log law}: \quad \log_axy=\log_ax + \log_ay[/tex]

[tex]\implies \sf \log_b5+\log_b5+\log_b3[/tex]

Substitute the given values to solve:

[tex]\implies \sf 1.609 + 1.609 + 1.099=4.317[/tex]

Question 2

Given:

  [tex]\sf \ln(4)=1.3863[/tex]

  [tex]\sf \ln(5)=1.6094[/tex]

To evaluate ln(20) replace 20 with (4 × 5):

[tex]\implies \sf \ln (4 \cdot 5)[/tex]

[tex]\textsf{Apply the Product log law}: \quad \ln xy=\ln x + \ln y[/tex]

[tex]\implies \sf \ln (4)+\ln (5)[/tex]

Substitute the given values to solve:

[tex]\implies \sf 1.3863+1.6094=2.9957[/tex]

I REQUIRE HELP IMMEDIATELY

Answers

All the equations have been solved

What is a Quadratic Equation ?

A quadratic equation is what can be represented by

ax² +bx+c = f(x)

It is given to solve the given quadratic equations

1.  x² = 25

x = ± 5

2. 16x² = 25

x² = 25/16

x = ± 5/4

3. (3x+2)²= 25

9x² +4+12x = 25

9x² +12x -21 = 0

3x(3x+7) -3(3x+7) = 0

x =1

x = -7/3

4. x² = 11x

x² -11x = 0

x (x-11) = 0

x = 0, 11

5. 3x²= 14x

3x² -14x = 0

x (3x - 14) = 0

x = 0 , 14/3

6. x²+2x = 3

x²+2x -3 = 0

x (x+3) -1(x+3) = 0

x = 1 ,-3

7.x²+2x -5 = 0

determined from the formula for the roots

x=−1±√6

x=1.44949

x=−3.44949

To know more about Quadratic Equation

https://brainly.com/question/2263981

#SPJ1

Geometry question! Please help thanks!

Answers

The measure of the angles are Z = 90, X = 90, V = 90, Y = ∠20, W = ∠8 and W = ∠20

How to solve the angles?

The sum of angles on a straight line is 180 degrees.

This means that:

Z + 90 = 180

X + 90 = 180

V + 90 = 180

Solve for the variables

Z = 90;

X = 90

V = 90

Also, alternate angles are equal.

This means that:

Y = ∠20

W = ∠8

This is so because the above angles are alternate angles.

Lastly, corresponding angles are equal.

So, we have:

W = ∠20

Read more about angles at:

https://brainly.com/question/14362353

#SPJ1

A local dry cleaner interviewed 17 candidates for the positions of the cashier, washer, salesperson, and delivery driver. How many ways can they fill the 4 positions?


HELP PLSPLSPLS

Answers

Using the permutation formula, it is found that there are 57,120 ways to fill the 4 positions.

There are different roles, hence the order is important, which means that the permutation formula is used.

What is the permutation formula?

The number of possible permutations of x elements from a set of n elements is given by:

[tex]P_{(n,x)} = \frac{n!}{(n-x)!}[/tex]

In this problem, 4 roles are chosen from a set of 17, hence the number of ways is given by:

[tex]P_{17,4} = \frac{17!}{13!} = 57120[/tex].

More can be learned about the permutation formula at https://brainly.com/question/25925367

#SPJ1

How do I solve for x?

Answers

Answer:

3

Step-by-step explanation:

1: Turn the mixed fraction into improper.

2: Multiply 3/5 to each side

3: The you get x = 3

You need to multiply the fraction instead of division because multiplying by the reciprocal is the same as dividing the normal fraction.

[tex]\huge\text{Hey there!}[/tex]


[tex]\mathsf{5 = 1\dfrac{2}{3}x}[/tex]

[tex]\mathsf{1\dfrac{2}{3}x = 5}[/tex]

[tex]\mathsf{\dfrac{1\times3 + 2}{3}x = 5}[/tex]

[tex]\mathsf{\dfrac{3 + 2}{3}x = 5}[/tex]

[tex]\mathsf{\dfrac{5}{3}x = 5}[/tex]

[tex]\large\textbf{MULTIPLY }\rm{\bf \dfrac{3}{5}}\large\textbf{ to BOTH SIDES}[/tex]

[tex]\mathsf{{\dfrac{3}{5}\times\dfrac{5}{3}x= \dfrac{3}{5}\times 5}}[/tex]

[tex]\large\textbf{SIMPLIFY IT!}[/tex]

[tex]\mathsf{x = \dfrac{3}{5}\times5}[/tex]

[tex]\mathsf{x = \dfrac{3}{5}\times\dfrac{5}{1}}[/tex]

[tex]\mathsf{x = \dfrac{3\times5}{5\times1}}[/tex]

[tex]\mathsf{x = \dfrac{15}{5}}[/tex]

[tex]\mathsf{x = 15\div5}[/tex]

[tex]\mathsf{x = 3}[/tex]

[tex]\huge\text{Therefore, your answer should be: \boxed{\mathsf{x = 3}}}\huge\checkmark[/tex]


[tex]\huge\text{Good luck on your assignment \& enjoy your day!}[/tex]


~[tex]\frak{Amphitrite1040:)}[/tex]

If the legs of a right triangle are 8 and 10, find the length of the hypotenuse.

Answers

Answer:

Given :-

Two legs of a rt. triangle : 8 and 10 respectively.

To find :-

Length of the hypotenuse

Solution :-

Using Pythagorean Theorem;

(hypotenuse)² = (8)² + (10)²

= 64 + 100

hypotenuse = √164

= 12.8 units

Answer:

12.80

Step-by-step explanation:

We have to use the Pythagoras theorem to find the value of the hypotenuse.

Let the value of the hypotenuse be c.

Let us use the below formula to find the length of the hypotenuse.

a² + b² = c²

First, let us replace a and b with 8 and 10 ( the lengths of the legs of the right triangle) with a and b.

a² + b² = c²

8² + 10² = c²

64 + 100 = c²

And now let us solve the above and find the value of c.

164 = c²

√164 = c

12.80 = c

Therefore, the length of the hypotenuse is:

12.80 units

PWhat are the x- and y-intercepts of the line y = 4x + 12?
A. x-intercept = 3; y-intercept = -12
B. x-intercept = -3; y-intercept = 12
C. x-intercept = -12; y-intercept = 3
D. x-intercept = 12; y-intercept = -3
PLEASE HELP

Answers

Answer:

B

Step-by-step explanation:

when x is 0 y=4(0)+12 y=12

y is 0 y=4(-3) +12 = 12=12

Other Questions
In parallelogram MNPQ if m/PQM = 140find m/MNP. Which quantity could best be measured with a meter stick?A. the amount of matter in a rockB. the width of a wooden blockC. the volume of water in a cupD. the diameter of the Earth Describe the recommendations of the American Association of Colleges of Nursing in response to the health profession's level of diversity in 2004. Mr. Ananta Jalil has received a job offer from a large investment bank as an assistant to the vice president. His base salary will be $35,000. He will receive his first annual salary payment one year from the day he begins to work. In addition, he will get an immediate $10,000 bonus for joining the company. His salary will grow at 4 percent each year. Mr. Jalil is expected to work for the next 5 years. What is the present value of the offer if the discount rate is 12 percent? [Hints: Mixed cash flow] The Volume of a cube depends on the length of its sides.This can be written in function notation as v(s).What is the best interpretation of V(3)=27 A co-worker has filed a document where you can access it if needed. This allows you to:Question options:copy the document in case of an emergency so both you and your supervisor will be able to access it.purge yourself of paper.copy the document so your supervisor will be able to access it as well.None of the above Which is the closest synonym for the word "pursue?" Carefully follow the steps to find the solution to the three equation system.1.2x+y+3:= 122. x-2y+z=-53.5.x- y+ 2z = 5a. Use equations 2 and 3 and eliminate the z by multiplication and addition, creating a new equation with only two variables.b. Use equations 1 and 2 and eliminate the z by multiplication and addition, creating a second equation with only two variables.c. Use the two new equations, and eliminate the x-variable by multiplication and addition, finding the value for the y-variable.d. Substitute y-value in the second new equation and find the x-value.e. Substitute the z-and y-values into original equation 2 to find the z-value A cup of water is warmed from 21 C to 85 C. What is the difference between these two temperatures, in kelvins? We know the sun is primarily made from hydrogen and helium on the basis of its: ii. Earth's stratosphere Weiming receives a weekly pocket money of $28. If he decides to save 20% of it, find his saving in a year and spendings in a year who was squanto?!?!?!?!?! Why is it important for the scrum master to help the team focus on daily and iteration goals\ vinegar contains 5g of acetic in 100mL of solution so the (w/v)% concentration is: Question 15 (1 point)The Johnson family consists of Jack (father, husband), Jill (mother, wife), and twochildren, Tommy and Tina. What type of family is this?NuclearBinuclearCohabitatingStructural Which statements describe aspects of the relationship between sleepdeprivation and drug abuse? Select the two correct answers.A. Sleep deprivation can increase a teen's chance of drug abuse.B. Sleep deprivation and drug abuse are not related.C. Drug abuse usually reduces sleep problems.D. Drug abuse can lead to insomnia and further drug abuse.? Why is this image an example of an appeal to pathos? U.S. Enters World War I I WANT YOU O 32 USA O A. Because it refers to the government's authority to help establish credibility B. Because it supplies new information to appeal to a sense of logic C. Because it uses feelings of patriotism to appeal to our emotions D. Because it uses facts and data to offer a logical explanation The terms of a particular sequence are determined according to the following rule: If the value of a given term $t$ is an odd positive integer, then the value of the following term is $3t -9$; if the value of a given term $t$ is an even positive integer, then the value of the following term is $2t -7$. Suppose that the terms of the sequence alternate between two positive integers $(a, b, a, b, \dots )$. What is the sum of the two positive integers what are factors may affect in Planning process